Các lớp phân phối đóng dưới mức tối đa


11

Đặt là một lớp phân phối xác suất trên các thực không âm được tham số hóa bởi , sao cho Tôi tự hỏi những lớp phân phối đã biết nào được đóng khi lấy tối đa và, ví dụ như nếu và là độc lập thì .Qpp

Qp([0,))=1.
X1Qp1X2Qp2max(X1,X2)Qp3

2
Bạn đang tìm kiếm một đặc tính toán học của các lớp như vậy hoặc bạn đang tìm hiểu về những họ phân phối tham số thường được biết đến có thể có thuộc tính này?
whuber


@whuber Tất cả ba loại giá trị cực đoan đều hoạt động theo đối số tôi đã đưa ra dưới đây. Tôi không cho thấy rằng họ là những người duy nhất mặc dù.
Michael R. Chernick

Powerpoint của Stoev mà whuber trích dẫn cho thấy kết quả mà tôi đã đưa ra cho các bản phân phối mà llya đã giải mã được gọi là maxi-ổn định và định lý được trích dẫn trong bài trình bày thêm rằng chúng là những bản duy nhất.
Michael R. Chernick

@Michael Bạn có nhận thấy sự hạn chế đối với các giá trị không âm trong câu hỏi không? Điều đó loại trừ các phân phối giá trị cực đoan có hỗ trợ tích cực trên các thực tế tiêu cực.
whuber

Câu trả lời:


12

Dường như với tôi rằng đề xuất phân phối giá trị cực đoan thực sự trả lời một câu hỏi khác. Tôi sẽ chứng minh rằng bằng cách giải quyết trực tiếp câu hỏi này và cho thấy nó dẫn đến các bản phân phối không nằm trong số các loại giá trị cực đoan.

Hãy xem xét điều này từ các nguyên tắc đầu tiên. Ngay lập tức, từ các tiên đề về xác suất và định nghĩa của CDF, rằng phân phối tối đa của hai biến ngẫu nhiên độc lập với và có cho CDF của nó. Giả sử tồn tại một lớp phân phối được đóng dưới mức tối đa theo cặp; đó là,F1F2F1F2Ω={Fθ}

FθΩ, FϕΩ implies FθFϕΩ.

Thật thuận tiện khi lấy logarit, mở rộng (như trong các văn bản phân tích nâng cao của Rudin) các số thực bao gồm là nhật ký . Nhật ký của CDF của các biến ngẫu nhiên về cơ bản được hỗ trợ trên là (i) không tăng đơn điệu, (ii) bằng trên , (iii) có giới hạn đúng là và ( iii) iv) là cadlag. Từ quan điểm này, phải là tập con lồi của hình nón trong không gian các hàm cadlag trên . Để nó được tham số hóa chính xác, hình nón đó phải tạo ra một không gian con vectơ hữu hạn. Điều đó vẫn để lại rất nhiều khả năng.0[0,)(,0)0ΩR

Một số khả năng này được biết đến. Ví dụ, xem xét CDF của một biến thống nhất trên . CDF của nó bằng trên , khi và trên . Hình nón mà nó tạo ra là tập hợp các CDF có dạng[0,1]0(,0]x0x11[1,)

Fθ(x)=exp(θlog(x))=xθ,0<x<1

được tham số hóa bởi . Rõ ràng tối đa của bất kỳ hai biến ngẫu nhiên độc lập nào có phân phối trong họ này cũng có phân phối trong họ này (tham số của chúng chỉ đơn giản là thêm). Nếu chúng ta muốn, chúng ta có thể giới hạn ở một tập hợp con lồi có dạng và vẫn có một gia đình đóng tối đa. Xin lưu ý rằng, không có thành viên nào trong gia đình này là một phân phối giá trị cực đoan.θ>0{Fθ|θθ0}

Công thức này bao gồm các phân phối rời rạc (rõ ràng không nằm trong số ba loại phân phối giá trị cực trị). Chẳng hạn, hãy xem xét các phân phối được hỗ trợ trên các số tự nhiên mà xác suất được đưa ra bởi0,1,2,,k,

Prθ(k)=θ1/(k+1)θ1/k

(lấy khi ), được tham số hóa bằng . Bằng cách xây dựng, CDF , từ đó nó đi theoθ1/k=0k=00<θ<1Fθ(k)=θ1/(k+1)

Fθ(k)Fϕ(k)=θ1/(k+1)ϕ1/(k+1)=(θϕ)1/(k+1),

và bởi vì các giả định ngụ ý , điều này cho thấy gia đình được đóng dưới cực đại cặp.0<θϕ<1

Tôi hy vọng rằng phân tích này và hai ví dụ này cho thấy, trái với ý kiến ​​được đưa ra trong một nhận xét, cách tiếp cận bắt đầu với một số lượng hữu hạn các CDF được chọn tốt và đóng chúng theo mức tối đa theo cặp (nghĩa là hình thành các hình nón của chúng trong một không gian vectơ có liên quan phù hợp) không chỉ mang tính xây dựng mà còn mang lại các lớp phân phối thú vị và có khả năng hữu ích.


3
+1 cho phân tích này và kiểm tra việc giải thích các phân phối giá trị cực đoan.

1
@whuber: cảm ơn bạn rất nhiều vì đã chú ý đến vấn đề này, tôi thực sự không mong đợi nhiều câu trả lời hay (và tôi sẽ chào mọi người đã trả lời). Cấu trúc hình nón (hoặc, nửa nhóm) mà bạn đã đưa ra thực sự đúng: nếu là bất kỳ họ phân phối nào thì việc đóng (wrt ) có mọi phần tử của biểu mẫu nơi và . Thật không may, tôi nhận ra rằng dịch chuyển wrt đóng cũng cần thiết (nghĩa là nếu thì ). Tôi có nên hỏi một câu hỏi mới cho việc này? max ( F α 1 θ 1 × × F α n θ n ) α i0 n N F ( x ) Ohm F ( x - một ) OhmFθmax(Fθ1α1××Fθnαn)αi0nNF(x)ΩF(xa)Ω
Ilya

1
Đó chắc chắn là một sự phức tạp, Ilya. Nhưng trước khi bạn thay đổi bất cứ điều gì hoặc đăng bất kỳ câu hỏi mới nào, vui lòng xem xét cách bạn sẽ điều hòa yêu cầu đóng ca làm việc với yêu cầu (dường như mâu thuẫn) rằng tất cả các biến đều có hỗ trợ không âm! (Tôi đoán bạn sẽ cần phải hạn chế các giá trị có thể có của .)a
whuber

Không liên quan đến câu hỏi này, nhưng tìm kiếm các ví dụ về các gia đình ổn định theo sản phẩm.
Vincent Granville

1
@Vincent Để bắt đầu, hãy xem xét bất kỳ họ biến ngẫu nhiên đóng phụ thuộc nào và lũy thừa chúng. Đối với một gia đình giàu có hơn, nhân bất kỳ biến nào trong số đó với biến độc lập Rademacher (thu được các biến được hỗ trợ trên toàn bộ dòng thực chứ không chỉ là các số dương). U
whuber

10

Lưu ý: Câu trả lời này giả định các biến được phân phối giống hệt nhau , không chỉ phân phối theo cùng một lớp.

Đó sẽ là những phân phối giá trị cực đoan . Có ba trong số chúng, như chúng thường được trình bày, tương ứng với ba bộ điều kiện trên phân phối cơ bản mà phân phối giới hạn tối đa được tìm thấy. Họ đang đóng cửa dưới mức tìm kiếm tối đa, đó là những gì bạn muốn.

Sao chép nhiều hơn hoặc ít hơn từ một phiên bản cũ của Phương pháp phân tích thống kê về độ tin cậy và dữ liệu cuộc sống (Mann, Schafer, Singpurwalla),

Loại I: FX(n)(x)=exp{exp[xγα]}, <x<, α>0

Loại II: FX(n)(x)=exp{(xγα)β}, xγ, α,β>0

Loại III: FX(n)(x)=exp{[(xγα)β]}. xγ, α,β>0

Chỉnh sửa: Đọc các bình luận, mở rộng câu trả lời này để làm cho một câu trả lời được cải thiện rất nhiều và đầy đủ hơn cho câu hỏi này!


3
+1 Nhưng loại I và III không áp dụng cho câu hỏi.
whuber

Hoàn toàn đúng (+1), tôi đã trả lời một câu hỏi tổng quát hơn mà không giải thích sự khác biệt. Ngoài ra, tôi nên mô tả sự bình thường hóa phải xảy ra để ngăn ngừa sự thoái hóa, như bạn đã làm trong bình luận của mình cho câu trả lời của MC dưới đây. Dạy tôi viết những câu trả lời này khi tôi sắp ra khỏi cửa! (tốt, có lẽ không ... :)
jbowman

1
@whuber Tôi có thể yêu cầu một cái gì đó hiển nhiên, nhưng, nó là sự thật rằng nếu và và họ là độc lập, sau đó ? X1Frechet(α1,β1)X2Frechet(α2,β2)max(X1,X2)Frechet(α3,β3)

2
Đó là một câu hỏi hay, @Procrastinator. Tôi không thể nghĩ ra bất kỳ lý do nào khiến kết quả như vậy là đúng, vì vậy tôi đã mô phỏng 1.000.000 giá trị iid từ Frechet và 1.000.000 giá trị iid từ Frechet và tính cực đại theo cặp của chúng. Các kết quả không thể phù hợp - thậm chí không xấp xỉ - bởi bất kỳ phân phối Frechet . Bạn cần cả ba tham số (bao gồm cả tham số vị trí) để đóng họ này dưới cực đại. Sau đó - mô phỏng một đối số (không đầy đủ) trong câu trả lời của Michael Chernick - bạn có thể hiển thị phải được thay đổi tỷ lệ Frechet. (3,1)(10,1)(α,β)max(X1,X2)
whuber

Câu trả lời này không chính xác. Định lý giá trị cực trị áp dụng khi phân phối của các biến giống nhau , nhưng câu hỏi nói rằng chúng chỉ phải thuộc về cùng một lớp (chúng có thể có các tham số khác nhau).
user76284

0

Jbowman đánh tôi để trả lời. Một lời giải thích cho lý do tại sao chúng hoạt động là Định lý của Gnedenko nói rằng nếu là một chuỗi gồm biến ngẫu nhiên phân phối độc lập trong các điều kiện nhất định trên đuôi của phân phối hội tụ đến 1 trong ba loại mà jbowman liệt kê trong câu trả lời của anh ta. Bây giờ vì mọi phân phối loại I, loại II hoặc loại III đều có thể được biểu thị dưới dạng giới hạn tối đa của chuỗi iid, nếu nói loại I và là phân phối giới hạn của vì có xu hướng vô cùng và cũng là loại I và là giới hạn củaX1,,XnnMn=max(X1,X2,,Xn)G1Mn=max(X1,X2,,Xn)nG2Nn=max(Y1,Y2,dotsc,Yn)sau đó nói và là phân phối giới hạn khi tiếp cận vô hạn cho thì sẽ là loại I và là phân phối tối đa của rv với phân phối và phân phối khác với phân phối và do đó loại I được đóng dưới mức tối đa hóa. Đối số tương tự làm việc cho loại II và loại III.Vn=max(Mn,Nn)G3nVnG3G1G2


2
Đối với các bản phân phối không giới hạn, mức tối đa không hội tụ: nó phân kỳ với . Như với CLT, cần phải chuẩn hóa thích hợp. (Đây là lý do tại sao nó là điều cần thiết để bao gồm vị trí và quy mô các thông số trong các gia đình.) Giấy cổ điển Gnedenko về đề tài này bắt đầu (nếu tôi nhớ chính xác) bằng cách đòi hỏi một loạt các hệ số affine thể được tìm thấy như vậy hội tụ. Sau khi thiết lập điều này, anh ta sau đó có được các hình thức phân phối hạn chế có thể. a n , b n a M n + b nnan,bnaMn+bn
whuber

Trong tất cả các trường hợp tôi nên nói bình thường hóa thích hợp. Cảm ơn. Ngay cả trong trường hợp bị ràng buộc, bạn phải bình thường hóa để đạt được giới hạn (tôi nghĩ, tôi nên nhớ điều này, luận án của tôi là cực đoan! Nhưng 34 năm trước)
Michael R. Chernick

3
Cũng lưu ý rằng các phân phối giá trị cực đoan không trả lời thấu đáo câu hỏi. (Đây không phải là chỉ trích, đây chỉ là một quan sát.) Ví dụ: giới hạn ở các số tự nhiên, chúng ta có thể định nghĩa là phân phối đồng đều trên . Lớp này được đóng dưới mức tối đa ( ), nhưng không có thành viên nào của nó là phân phối giá trị cực trị. Q p [ p , p + 1 ] max ( Q p , Q r ) ~ Q max ( p , r )pQp[p,p+1]max(Qp,Qr)Qmax(p,r)
whuber

@whuber cả ba loại đều là trường hợp không giới hạn nhưng loại III đuôi ngắn bao gồm các trường hợp giới hạn như phân phối đồng đều. Với U [0,1], P [Mn <= 1-x / n] hội tụ thành exp (-x) vì P [Mn <= 1-x / n] = (1-x / n) ^ n.
Michael R. Chernick

3
Câu trả lời của bạn dường như không liên quan trong ví dụ tôi đưa ra, Michael. Điểm khác biệt là câu hỏi này không hỏi về các chuỗi có thể đếm được của các biến iid hoặc thậm chí là chuỗi có thể đếm được của bất cứ thứ gì; nó chỉ hỏi về việc đóng dưới các cặp biến thường có các phân phối khác nhau . (Nhưng bây giờ tôi thấy có một lỗ hổng trong ví dụ của mình: tối đa khi không còn đồng đều, vì vậy tôi sẽ phải phóng to gia đình một cách thích hợp để bao gồm tối đa nhiều đồng phục iid tùy ý.)p=r
whuber
Khi sử dụng trang web của chúng tôi, bạn xác nhận rằng bạn đã đọc và hiểu Chính sách cookieChính sách bảo mật của chúng tôi.
Licensed under cc by-sa 3.0 with attribution required.